QFT: Lorentz Trans+ Field infinitesimal variation

In summary: I did not find anything else than the usual ##\phi(x)## and not ##\phi(x')##. I will try to do it with the form you suggested (i.e. using ##\phi(x')##). Thank you for the help!In summary, the conversation revolved around the computation of the infinitesimal variation of a field under Lorentz transformation, specifically the term ##\Delta_a=\phi_a'(x)-\phi_a(x)## appearing in Noether's theorem. The conversation also discussed the 4-vector representation of the Lorentz transform, the Lorentz generators, and the definition of a tensor or spinor field. The final conclusion was to use the local representation of the
  • #1
Mishra
55
1
Hello,

I do not understand how to compute the infinitesimal variation of the field at fixed coordinates; under lorentz transformation . I am doing something wrong regarding the transformation of the ##x## coordinate.

I am looking for: ##\Delta_a=\phi_a'(x)-\phi_a(x)##, variation appearing in Noether's theorem ##j^{\mu}_{\nu}=\frac{\partial L}{\partial \partial\phi}\Delta_a - \epsilon^{\mu}_{\nu}L##

Lorentz transform:
(1): ##x^{\mu}\rightarrow x'^{\mu}=\Lambda^{\mu}_{\nu}x^{\nu}=x^{\mu}-\omega^{\mu}_{\nu}x^{\nu}## 4vector representation
(2): ##\phi_a(x)\rightarrow\phi_a'(x')=M^{b}_a\phi_b(x')## any field representation

From (2) we get:
##\phi_a'(x)=M^{b}_a\phi_b(\Lambda^{-1}x)##

Therefore:
##\Delta_a=\phi_a'(x)-\phi_a(x)## becomes:
(3): ##\Delta_a=M^{b}_a\phi_b(\Lambda^{-1}x)-\phi_a(x)##

Recalling the Lorentz generators:
(4): ##M=exp(-\frac{i}{2}\omega_{\mu\nu}J^{\mu\nu})=1-\frac{i}{2}\omega_{\mu\nu}J^{\mu\nu}##

I can now plug the generator into into (3):
##\Delta_a= (1-\frac{i}{2}\omega_{\mu\nu}J^{\mu\nu})^{b}_a\phi_b(\Lambda^{-1}x)-\phi_a(x)##
##\Delta_a= (1-\frac{i}{2}\omega_{\mu\nu}J^{\mu\nu})^{b}_a\phi_b(x^{\mu}+\omega^{\mu}_{\nu}x^{\nu})-\phi_a(x)##

Rearranging the terms I end up with:
##\Delta_a=\phi_a(x^{\mu}+\omega^{\mu}_{\nu}x^{\nu})-\phi_a(x) - (\frac{i}{2}\omega_{\mu\nu}J^{\mu\nu})^{b}_a\phi_b(x^{\mu}+\omega^{\mu}_{\nu}x^{\nu})##

The first two terms are a simple derivative so:
##\Delta_a=-\omega^{\mu}_{\nu}x^{\nu}\partial_{\mu}\phi_a(x) - (\frac{i}{2}\omega_{\mu\nu}J^{\mu\nu})^{b}_a\phi_b(x^{\mu}+\omega^{\mu}_{\nu}x^{\nu})##

And this is where I fail. According to pretty much every book I could find I should simply have:
##\Delta_a=-\omega^{\mu}_{\nu}x^{\nu}\partial_{\mu}\phi_a(x) - (\frac{i}{2}\omega_{\mu\nu}J^{\mu\nu})^{b}_a\phi_b(x)##

How do I get rid of this? I am probably doing something wrong at the very beginning where I define either the transformation or the infinitesimal change of the field but I don't understand why... I think I do not understand how the transformation carries the field/coordinates.

Thanks!
VM
 
Physics news on Phys.org
  • #2
Mishra said:
Hello,
##\Delta_a=-\omega^{\mu}_{\nu}x^{\nu}\partial_{\mu}\phi_a(x) - (\frac{i}{2}\omega_{\mu\nu}J^{\mu\nu})^{b}_a\phi_b(x^{\mu}+\omega^{\mu}_{\nu}x^{\nu})##
Thanks!
VM
Why didn't you expand [itex]\varphi(x + \omega \cdot x)[/itex] in the last term? Do that and keep only linear terms in [itex]\omega[/itex], because, infinitesimally [itex]\omega^{2} \approx 0[/itex]:
[tex](\omega \cdot J)\varphi(x + \omega \cdot x) = (\omega \cdot J) \ \varphi(x) + (\omega \cdot J) \ \omega \cdot x \cdot \partial \varphi (x) \approx (\omega \cdot J) \ \varphi (x)[/tex]
 
  • Like
Likes vanhees71
  • #3
How dumb of me.

thank you very much!
 
  • #4
I would just have an other question. Is it correct to say that the transformation carries the field this way:
(1): ##x^{\mu}\rightarrow x'^{\mu}=\Lambda^{\mu}_{\nu}x^{\nu}=x^{\mu}-\omega^{\mu}_{\nu}x^{\nu}##
(2): ##\phi_a(x)\rightarrow\phi_a'(x')=M^{b}_a\phi_b(x')##

We then look at the variation at fixed coordinnates: ##\Delta_a=\phi_a'(x)-\phi_a(x)## by computing: ##\phi_a'(x)=M^{b}_a\phi_b(\Lambda^{-1}x)##.

I am asking this because I see a lot of these in various lecture notes:
##x^{\mu}\rightarrow x'^{\mu}=\Lambda^{\mu}_{\nu}x^{\nu}=x^{\mu}-\omega^{\mu}_{\nu}x^{\nu}##
##\phi_a(x)\rightarrow\phi_a'(x')=M^{b}_a\phi_b(x)## (unprimed ##x## here)

Is this just a common way of writting it (after the expansion you were mentionning) ?
 
  • #5
I'd say your own expression equals these textbook expressions. If you replace the unprimed x by (Lamda-1)x in your last expression, and also at the LHS, you see the equality.
 
  • #6
Mishra said:
I would just have an other question. Is it correct to say that the transformation carries the field this way:
(1): ##x^{\mu}\rightarrow x'^{\mu}=\Lambda^{\mu}_{\nu}x^{\nu}=x^{\mu}-\omega^{\mu}_{\nu}x^{\nu}##
(2): ##\phi_a(x)\rightarrow\phi_a'(x')=M^{b}_a\phi_b(x')##

We then look at the variation at fixed coordinnates: ##\Delta_a=\phi_a'(x)-\phi_a(x)## by computing: ##\phi_a'(x)=M^{b}_a\phi_b(\Lambda^{-1}x)##.

I am asking this because I see a lot of these in various lecture notes:
##x^{\mu}\rightarrow x'^{\mu}=\Lambda^{\mu}_{\nu}x^{\nu}=x^{\mu}-\omega^{\mu}_{\nu}x^{\nu}##
##\phi_a(x)\rightarrow\phi_a'(x')=M^{b}_a\phi_b(x)## (unprimed ##x## here)

Is this just a common way of writting it (after the expansion you were mentionning) ?
The latter is the correct definition of a tensor or spinor field (depending on the meaning of the latin indices and the matrix in your equation). This is what's called a local representation of the (proper orthochronous) Lorentz group.
 
  • #7
vanhees71 said:
The latter is the correct definition of a tensor or spinor field (depending on the meaning of the latin indices and the matrix in your equation). This is what's called a local representation of the (proper orthochronous) Lorentz group.

This is what I do not understand. When you apply the transformation to ##\phi(x)## why would you only end up with ##M\phi(x)## ?
It seems to me that the coordinates change too and I should get ##M\phi(x')## (where M is a non trivial representation acting on the field).

edit: Also, this would not be consistant with the computations in my original post no ?
 
  • #8
Mishra said:
This is what I do not understand. When you apply the transformation to ##\phi(x)## why would you only end up with ##M\phi(x)## ?
It seems to me that the coordinates change too and I should get ##M\phi(x')## (where M is a non trivial representation acting on the field).

edit: Also, this would not be consistant with the computations in my original post no ?

Think of [itex]x[/itex] and [itex]\bar{x} = \Lambda x[/itex] as two coordinate values of the same point [itex]P[/itex] of the space-time manifold. So, [itex]\varphi_{a}(x)[/itex] and [itex]\bar{\varphi}_{a}(\bar{x})[/itex] are the values of the field at [itex]P[/itex] as seen in the unbarred and barred systems respectively. The relation between the two is determined by the nature of the index [itex]a[/itex], i.e., the dimension of the representation space to which the field [itex]\varphi[/itex] belongs:
[tex]\bar{\varphi}_{a} (\bar{x}) = M_{ab}(\Lambda) \ \varphi_{b}(x) , \ \ \ \ \ \ \ \ (1)[/tex]
where [itex]M(\Lambda)[/itex] is the appropriate matrix representation of the Lorentz group:
[tex]M(\Lambda_{1}) M(\Lambda_{2}) = M(\Lambda_{1}\Lambda_{2}) .[/tex]
You can also rewrite (1) as
[tex]\bar{\varphi}_{a} (\bar{x}) = M_{ab}(\Lambda) \ \varphi_{b}(\Lambda^{-1}\bar{x}) ,[/tex]
and rename the coordinates as [itex]x[/itex]
[tex]\bar{\varphi}_{a} (x) = M_{ab}(\Lambda) \ \varphi_{b}(\Lambda^{-1}x) .[/tex]
Have a look at the PDH below.
 

Attachments

  • Noether Theorem.pdf
    359.9 KB · Views: 210
  • #9
Mishra said:
This is what I do not understand. When you apply the transformation to ##\phi(x)## why would you only end up with ##M\phi(x)## ?
It seems to me that the coordinates change too and I should get ##M\phi(x')## (where M is a non trivial representation acting on the field).

edit: Also, this would not be consistant with the computations in my original post no ?
No, the correct transformation is
$$\phi'(x')=\hat{M} \phi(x)=\hat{M} \phi(\Lambda^{-1} x').$$
This makes much sense, because the field describes a certain physical situation, and the transformation describes how its components change between two (inertial) observers. Each of them uses his space-time coordinates and his components of the field.

In the quantized version the transformation law reads
$$\hat{U} \hat{\phi}(x') \hat{U}^{-1}=\hat{M} \hat{\phi}(\Lambda^{-1} x').$$
 
  • #10
The more I think about it the more I realize I am missing something big here.

To me it would seem that the field and the coordinates are tranformed. if point P has coordinates ##x'=\Lambda x## in the two different frames (related by the transformation ##\Lambda##). Then the field ##\phi(x)## seen at ##x##, is transformed to the field ##\phi'(x')## seen at ##x'##.
The "old field" in the old coordinates is transformed into the new field in the new coordinates.

What am I missing here ?

edit:
Just in case, I also found a mistake in my original post. I Somehow found the correct ##\phi'(x)-\phi(x)=M\phi(\Lambda^{-1}x)-\phi(x)## while having equation (2) wrong.
 
  • #11
I thinks, you don't miss anything there. Why do you think, you did?
 
  • #12
Well, in such case I should have:

Old field at x = new field at x'
ie: ##\phi(x)=M\phi(x')##
 
  • #13
Why that? Take the most simple example of a scalar field first, say the temperature field of the air in your room. It's defined by the temperature values at any point in your room at a given time, which defines a scalar field ##T(x)##. Now suppose Alice is moving with some constant velocity relative to your room. She'll measure the temperature too, and according to the rules, how temperature (nowadays!) is defined, she does this with a thermometer momentarily at rest with the air in your room. Then she'll get the same values at any given point, but she's using her own space-time coordinates. So she will get the field
$$T'(x')=T(x).$$
Now you have
$$x' =\Lambda x,$$
with a Lorentz matrix ##\Lambda##. Then the above equation reads
$$T'(x')=T(x)=T(\Lambda^{-1} x').$$
Of course, now you can write any variable for the space-time positions, e.g., ##x'=y##:
$$T'(y)=T(\Lambda^{-1} y).$$
Of course, now you can also set ##y=x##, and you have
$$T'(x)=T(\Lambda^{-1} x).$$
That's a bit confusing in the beginning (at least it confused me a lot when I started to learn about Noether's theorem in field theory), but it's just an issue mathematical notation. Note that in the latter form we have only one set of space-time coordinates on each side, while in the defining equation, we have two sets of space-time coordinates, yours (we then called ##x##) and Alice's (we then called ##x'##).

The latter form is important, because it explains why the definition of the unitary transformations of the Poincare group in the quantized theory look as they do, i.e., for a scalar field and a boost
$$\hat{\phi}'(x)=\hat{U}(\Lambda) \hat{\phi}(x) \hat{U}^{-1}(\Lambda) = \hat{U}(\Lambda) \hat{\phi}(x) \hat{U}^{\dagger}(\Lambda) = \hat{\phi}(\Lambda^{-1} x).$$
 
  • Like
Likes samalkhaiat
  • #14
Mishra said:
Well, in such case I should have:

Old field at x = new field at x'
ie: ##\phi(x)=M\phi(x')##
No, the functional change in the field is given by [itex]\bar{\varphi} = M \varphi[/itex]. In group theory language, this means that [itex]\varphi[/itex] and [itex]\bar{\varphi}[/itex] belong to the same representation space (same multiplet). Now, evaluate both side at the point [itex]P[/itex], i.e., [itex]\bar{\varphi} (P)= M \varphi (P)[/itex]. If, you are sitting in the [itex]\bar{S}[/itex]-system, you take [itex]\bar{x}[/itex] to be the coordinates of the point [itex]P[/itex], while, your friend at the [itex]S[/itex]-system uses [itex]x[/itex], so [itex]\bar{\varphi} (\bar{x})= M \varphi (x)[/itex]. The transformation matrix [itex]M^{a}{}_{b}[/itex] depends on the nature of the field [itex]\varphi_{a}[/itex] ,i.e., the representation space to which the field belong. For example, a scalar field transforms in the trivial (one dimensional) representation, so [itex]M^{a}{}_{b} = \delta^{a}_{b}[/itex], implying that [itex]\bar{\varphi}_{a}(\bar{x}) = \varphi_{a}(x)[/itex].
 
  • Like
Likes vanhees71
  • #15
I got it now!
You transform the old field you get a new one, in a new frame. This was so simple...

Thanks a lot!
 
  • #16
To make it less simple you could now consider passive and active interpretations, and in the context of general relativity. Ho ho ho, merry christmas :P
 

1. What is QFT?

Quantum field theory (QFT) is a theoretical framework used to describe the behavior of subatomic particles and their interactions. It combines the principles of quantum mechanics and special relativity to explain the dynamics of particles and fields at the smallest scales.

2. What is the Lorentz transformation in QFT?

The Lorentz transformation is a mathematical tool used to describe how physical quantities, such as space and time, appear to change when observed from different reference frames in special relativity. In QFT, it is used to transform fields and their corresponding equations to maintain their form in different frames of reference.

3. How does QFT handle infinitesimal variations in fields?

In QFT, fields are described as continuous functions that can vary infinitesimally in space and time. These variations are handled using techniques from calculus, such as derivatives and integrals, to calculate the behavior of the fields and their interactions with other particles.

4. What is the significance of infinitesimal variations in QFT?

Infinitesimal variations in fields are crucial in QFT as they allow for the precise calculation of particle interactions and the prediction of their behavior. Without considering these small variations, the theory would not accurately describe the dynamics of particles at the quantum level.

5. What are the applications of QFT?

QFT has many applications in modern physics, including particle physics, solid-state physics, and cosmology. It has been used to develop the Standard Model of particle physics, which explains the fundamental particles and their interactions, and to study the behavior of matter at extreme conditions, such as in the early universe or in black holes.

Similar threads

Replies
5
Views
373
  • Quantum Physics
Replies
2
Views
999
Replies
6
Views
1K
Replies
24
Views
2K
  • Quantum Physics
Replies
3
Views
1K
Replies
5
Views
788
  • Quantum Physics
Replies
1
Views
605
  • Quantum Physics
Replies
3
Views
387
  • Quantum Physics
Replies
3
Views
2K
Replies
1
Views
847
Back
Top